Is ∅ ⊈ { ∅, 1, 2 } False?












0












$begingroup$


Is this ∅ ⊈ { ∅, 1, 2 } true or false ?



Also, I am confuse since this { ∅, 1, 2 } has already contain a , does it still contain another meaning like : { ∅, ∅, 1, 2 } ?



Is ∅ ∈ { ∅, 1, 2 } true ? & {∅} ∈ { ∅, 1, 2 } false ?










share|cite|improve this question









$endgroup$












  • $begingroup$
    It is trivially false since empty set is the subset of any set. This comes from the fact that empty set does not contain any element.
    $endgroup$
    – Le Anh Dung
    Jan 21 at 5:47






  • 2




    $begingroup$
    Don't confuse $in$ with $subseteq$.
    $endgroup$
    – Cheerful Parsnip
    Jan 21 at 5:55










  • $begingroup$
    $emptyset subset A $ is always true no matter what set $A $ is.
    $endgroup$
    – fleablood
    Jan 21 at 7:21
















0












$begingroup$


Is this ∅ ⊈ { ∅, 1, 2 } true or false ?



Also, I am confuse since this { ∅, 1, 2 } has already contain a , does it still contain another meaning like : { ∅, ∅, 1, 2 } ?



Is ∅ ∈ { ∅, 1, 2 } true ? & {∅} ∈ { ∅, 1, 2 } false ?










share|cite|improve this question









$endgroup$












  • $begingroup$
    It is trivially false since empty set is the subset of any set. This comes from the fact that empty set does not contain any element.
    $endgroup$
    – Le Anh Dung
    Jan 21 at 5:47






  • 2




    $begingroup$
    Don't confuse $in$ with $subseteq$.
    $endgroup$
    – Cheerful Parsnip
    Jan 21 at 5:55










  • $begingroup$
    $emptyset subset A $ is always true no matter what set $A $ is.
    $endgroup$
    – fleablood
    Jan 21 at 7:21














0












0








0





$begingroup$


Is this ∅ ⊈ { ∅, 1, 2 } true or false ?



Also, I am confuse since this { ∅, 1, 2 } has already contain a , does it still contain another meaning like : { ∅, ∅, 1, 2 } ?



Is ∅ ∈ { ∅, 1, 2 } true ? & {∅} ∈ { ∅, 1, 2 } false ?










share|cite|improve this question









$endgroup$




Is this ∅ ⊈ { ∅, 1, 2 } true or false ?



Also, I am confuse since this { ∅, 1, 2 } has already contain a , does it still contain another meaning like : { ∅, ∅, 1, 2 } ?



Is ∅ ∈ { ∅, 1, 2 } true ? & {∅} ∈ { ∅, 1, 2 } false ?







elementary-set-theory






share|cite|improve this question













share|cite|improve this question











share|cite|improve this question




share|cite|improve this question










asked Jan 21 at 5:44









J.SJ.S

555




555












  • $begingroup$
    It is trivially false since empty set is the subset of any set. This comes from the fact that empty set does not contain any element.
    $endgroup$
    – Le Anh Dung
    Jan 21 at 5:47






  • 2




    $begingroup$
    Don't confuse $in$ with $subseteq$.
    $endgroup$
    – Cheerful Parsnip
    Jan 21 at 5:55










  • $begingroup$
    $emptyset subset A $ is always true no matter what set $A $ is.
    $endgroup$
    – fleablood
    Jan 21 at 7:21


















  • $begingroup$
    It is trivially false since empty set is the subset of any set. This comes from the fact that empty set does not contain any element.
    $endgroup$
    – Le Anh Dung
    Jan 21 at 5:47






  • 2




    $begingroup$
    Don't confuse $in$ with $subseteq$.
    $endgroup$
    – Cheerful Parsnip
    Jan 21 at 5:55










  • $begingroup$
    $emptyset subset A $ is always true no matter what set $A $ is.
    $endgroup$
    – fleablood
    Jan 21 at 7:21
















$begingroup$
It is trivially false since empty set is the subset of any set. This comes from the fact that empty set does not contain any element.
$endgroup$
– Le Anh Dung
Jan 21 at 5:47




$begingroup$
It is trivially false since empty set is the subset of any set. This comes from the fact that empty set does not contain any element.
$endgroup$
– Le Anh Dung
Jan 21 at 5:47




2




2




$begingroup$
Don't confuse $in$ with $subseteq$.
$endgroup$
– Cheerful Parsnip
Jan 21 at 5:55




$begingroup$
Don't confuse $in$ with $subseteq$.
$endgroup$
– Cheerful Parsnip
Jan 21 at 5:55












$begingroup$
$emptyset subset A $ is always true no matter what set $A $ is.
$endgroup$
– fleablood
Jan 21 at 7:21




$begingroup$
$emptyset subset A $ is always true no matter what set $A $ is.
$endgroup$
– fleablood
Jan 21 at 7:21










3 Answers
3






active

oldest

votes


















2












$begingroup$

$$oslashsubset{oslash,1,2}$$ and $$oslashin{oslash,1,2}.$$
Also, $${oslash}subset{oslash,1,2}.$$






share|cite|improve this answer









$endgroup$













  • $begingroup$
    Sorry, why is {⊘} ⊂ {⊘,1,2} ?
    $endgroup$
    – J.S
    Jan 21 at 5:53










  • $begingroup$
    Because it's an element of the set. Don't you write $$ {1} subset {1, 2, 3} $$??
    $endgroup$
    – Anik Bhowmick
    Jan 21 at 5:54












  • $begingroup$
    @J.S Because $oslashin{oslash,1,2}.$
    $endgroup$
    – Michael Rozenberg
    Jan 21 at 5:54



















0












$begingroup$

No, not like that. Since null set is subset of all sets, the question in your title is valid. At least I think so.






share|cite|improve this answer









$endgroup$





















    0












    $begingroup$

    $neg (Asubset B)$ iff there exists a member of $A$ that is not a member of $B$.



    Consider the case $A=phi.$



    Since $phi$ has no members, there cannot exist a member of $phi$ that fails to belong to $B$. Therefore $neg (neg (phisubset B))$, equivalently $phisubset B.$ For $any $ $B$.






    share|cite|improve this answer









    $endgroup$













      Your Answer





      StackExchange.ifUsing("editor", function () {
      return StackExchange.using("mathjaxEditing", function () {
      StackExchange.MarkdownEditor.creationCallbacks.add(function (editor, postfix) {
      StackExchange.mathjaxEditing.prepareWmdForMathJax(editor, postfix, [["$", "$"], ["\\(","\\)"]]);
      });
      });
      }, "mathjax-editing");

      StackExchange.ready(function() {
      var channelOptions = {
      tags: "".split(" "),
      id: "69"
      };
      initTagRenderer("".split(" "), "".split(" "), channelOptions);

      StackExchange.using("externalEditor", function() {
      // Have to fire editor after snippets, if snippets enabled
      if (StackExchange.settings.snippets.snippetsEnabled) {
      StackExchange.using("snippets", function() {
      createEditor();
      });
      }
      else {
      createEditor();
      }
      });

      function createEditor() {
      StackExchange.prepareEditor({
      heartbeatType: 'answer',
      autoActivateHeartbeat: false,
      convertImagesToLinks: true,
      noModals: true,
      showLowRepImageUploadWarning: true,
      reputationToPostImages: 10,
      bindNavPrevention: true,
      postfix: "",
      imageUploader: {
      brandingHtml: "Powered by u003ca class="icon-imgur-white" href="https://imgur.com/"u003eu003c/au003e",
      contentPolicyHtml: "User contributions licensed under u003ca href="https://creativecommons.org/licenses/by-sa/3.0/"u003ecc by-sa 3.0 with attribution requiredu003c/au003e u003ca href="https://stackoverflow.com/legal/content-policy"u003e(content policy)u003c/au003e",
      allowUrls: true
      },
      noCode: true, onDemand: true,
      discardSelector: ".discard-answer"
      ,immediatelyShowMarkdownHelp:true
      });


      }
      });














      draft saved

      draft discarded


















      StackExchange.ready(
      function () {
      StackExchange.openid.initPostLogin('.new-post-login', 'https%3a%2f%2fmath.stackexchange.com%2fquestions%2f3081552%2fis-%25e2%2588%2585-%25e2%258a%2586%25cc%25b8-%25e2%2588%2585-1-2-false%23new-answer', 'question_page');
      }
      );

      Post as a guest















      Required, but never shown

























      3 Answers
      3






      active

      oldest

      votes








      3 Answers
      3






      active

      oldest

      votes









      active

      oldest

      votes






      active

      oldest

      votes









      2












      $begingroup$

      $$oslashsubset{oslash,1,2}$$ and $$oslashin{oslash,1,2}.$$
      Also, $${oslash}subset{oslash,1,2}.$$






      share|cite|improve this answer









      $endgroup$













      • $begingroup$
        Sorry, why is {⊘} ⊂ {⊘,1,2} ?
        $endgroup$
        – J.S
        Jan 21 at 5:53










      • $begingroup$
        Because it's an element of the set. Don't you write $$ {1} subset {1, 2, 3} $$??
        $endgroup$
        – Anik Bhowmick
        Jan 21 at 5:54












      • $begingroup$
        @J.S Because $oslashin{oslash,1,2}.$
        $endgroup$
        – Michael Rozenberg
        Jan 21 at 5:54
















      2












      $begingroup$

      $$oslashsubset{oslash,1,2}$$ and $$oslashin{oslash,1,2}.$$
      Also, $${oslash}subset{oslash,1,2}.$$






      share|cite|improve this answer









      $endgroup$













      • $begingroup$
        Sorry, why is {⊘} ⊂ {⊘,1,2} ?
        $endgroup$
        – J.S
        Jan 21 at 5:53










      • $begingroup$
        Because it's an element of the set. Don't you write $$ {1} subset {1, 2, 3} $$??
        $endgroup$
        – Anik Bhowmick
        Jan 21 at 5:54












      • $begingroup$
        @J.S Because $oslashin{oslash,1,2}.$
        $endgroup$
        – Michael Rozenberg
        Jan 21 at 5:54














      2












      2








      2





      $begingroup$

      $$oslashsubset{oslash,1,2}$$ and $$oslashin{oslash,1,2}.$$
      Also, $${oslash}subset{oslash,1,2}.$$






      share|cite|improve this answer









      $endgroup$



      $$oslashsubset{oslash,1,2}$$ and $$oslashin{oslash,1,2}.$$
      Also, $${oslash}subset{oslash,1,2}.$$







      share|cite|improve this answer












      share|cite|improve this answer



      share|cite|improve this answer










      answered Jan 21 at 5:47









      Michael RozenbergMichael Rozenberg

      107k1894198




      107k1894198












      • $begingroup$
        Sorry, why is {⊘} ⊂ {⊘,1,2} ?
        $endgroup$
        – J.S
        Jan 21 at 5:53










      • $begingroup$
        Because it's an element of the set. Don't you write $$ {1} subset {1, 2, 3} $$??
        $endgroup$
        – Anik Bhowmick
        Jan 21 at 5:54












      • $begingroup$
        @J.S Because $oslashin{oslash,1,2}.$
        $endgroup$
        – Michael Rozenberg
        Jan 21 at 5:54


















      • $begingroup$
        Sorry, why is {⊘} ⊂ {⊘,1,2} ?
        $endgroup$
        – J.S
        Jan 21 at 5:53










      • $begingroup$
        Because it's an element of the set. Don't you write $$ {1} subset {1, 2, 3} $$??
        $endgroup$
        – Anik Bhowmick
        Jan 21 at 5:54












      • $begingroup$
        @J.S Because $oslashin{oslash,1,2}.$
        $endgroup$
        – Michael Rozenberg
        Jan 21 at 5:54
















      $begingroup$
      Sorry, why is {⊘} ⊂ {⊘,1,2} ?
      $endgroup$
      – J.S
      Jan 21 at 5:53




      $begingroup$
      Sorry, why is {⊘} ⊂ {⊘,1,2} ?
      $endgroup$
      – J.S
      Jan 21 at 5:53












      $begingroup$
      Because it's an element of the set. Don't you write $$ {1} subset {1, 2, 3} $$??
      $endgroup$
      – Anik Bhowmick
      Jan 21 at 5:54






      $begingroup$
      Because it's an element of the set. Don't you write $$ {1} subset {1, 2, 3} $$??
      $endgroup$
      – Anik Bhowmick
      Jan 21 at 5:54














      $begingroup$
      @J.S Because $oslashin{oslash,1,2}.$
      $endgroup$
      – Michael Rozenberg
      Jan 21 at 5:54




      $begingroup$
      @J.S Because $oslashin{oslash,1,2}.$
      $endgroup$
      – Michael Rozenberg
      Jan 21 at 5:54











      0












      $begingroup$

      No, not like that. Since null set is subset of all sets, the question in your title is valid. At least I think so.






      share|cite|improve this answer









      $endgroup$


















        0












        $begingroup$

        No, not like that. Since null set is subset of all sets, the question in your title is valid. At least I think so.






        share|cite|improve this answer









        $endgroup$
















          0












          0








          0





          $begingroup$

          No, not like that. Since null set is subset of all sets, the question in your title is valid. At least I think so.






          share|cite|improve this answer









          $endgroup$



          No, not like that. Since null set is subset of all sets, the question in your title is valid. At least I think so.







          share|cite|improve this answer












          share|cite|improve this answer



          share|cite|improve this answer










          answered Jan 21 at 5:47









          Anik BhowmickAnik Bhowmick

          583417




          583417























              0












              $begingroup$

              $neg (Asubset B)$ iff there exists a member of $A$ that is not a member of $B$.



              Consider the case $A=phi.$



              Since $phi$ has no members, there cannot exist a member of $phi$ that fails to belong to $B$. Therefore $neg (neg (phisubset B))$, equivalently $phisubset B.$ For $any $ $B$.






              share|cite|improve this answer









              $endgroup$


















                0












                $begingroup$

                $neg (Asubset B)$ iff there exists a member of $A$ that is not a member of $B$.



                Consider the case $A=phi.$



                Since $phi$ has no members, there cannot exist a member of $phi$ that fails to belong to $B$. Therefore $neg (neg (phisubset B))$, equivalently $phisubset B.$ For $any $ $B$.






                share|cite|improve this answer









                $endgroup$
















                  0












                  0








                  0





                  $begingroup$

                  $neg (Asubset B)$ iff there exists a member of $A$ that is not a member of $B$.



                  Consider the case $A=phi.$



                  Since $phi$ has no members, there cannot exist a member of $phi$ that fails to belong to $B$. Therefore $neg (neg (phisubset B))$, equivalently $phisubset B.$ For $any $ $B$.






                  share|cite|improve this answer









                  $endgroup$



                  $neg (Asubset B)$ iff there exists a member of $A$ that is not a member of $B$.



                  Consider the case $A=phi.$



                  Since $phi$ has no members, there cannot exist a member of $phi$ that fails to belong to $B$. Therefore $neg (neg (phisubset B))$, equivalently $phisubset B.$ For $any $ $B$.







                  share|cite|improve this answer












                  share|cite|improve this answer



                  share|cite|improve this answer










                  answered Jan 21 at 6:59









                  DanielWainfleetDanielWainfleet

                  35.3k31648




                  35.3k31648






























                      draft saved

                      draft discarded




















































                      Thanks for contributing an answer to Mathematics Stack Exchange!


                      • Please be sure to answer the question. Provide details and share your research!

                      But avoid



                      • Asking for help, clarification, or responding to other answers.

                      • Making statements based on opinion; back them up with references or personal experience.


                      Use MathJax to format equations. MathJax reference.


                      To learn more, see our tips on writing great answers.




                      draft saved


                      draft discarded














                      StackExchange.ready(
                      function () {
                      StackExchange.openid.initPostLogin('.new-post-login', 'https%3a%2f%2fmath.stackexchange.com%2fquestions%2f3081552%2fis-%25e2%2588%2585-%25e2%258a%2586%25cc%25b8-%25e2%2588%2585-1-2-false%23new-answer', 'question_page');
                      }
                      );

                      Post as a guest















                      Required, but never shown





















































                      Required, but never shown














                      Required, but never shown












                      Required, but never shown







                      Required, but never shown

































                      Required, but never shown














                      Required, but never shown












                      Required, but never shown







                      Required, but never shown







                      Popular posts from this blog

                      Can a sorcerer learn a 5th-level spell early by creating spell slots using the Font of Magic feature?

                      Does disintegrating a polymorphed enemy still kill it after the 2018 errata?

                      A Topological Invariant for $pi_3(U(n))$